Intégrale mercredi 3 août — Les-mathematiques.net The most powerful custom community solution in the world

Intégrale mercredi 3 août

Modifié (August 2022) dans Analyse
Bonjour
Calculer $$\int_{-1}^{1} \frac{\arccos(x)}{x^2+x+1}dx$$ Merci.
«1

Réponses

  • Un petit indice s'il te plaît ? 
  • Je pense à un truc mais je n'en suis pas sûr... 
  • Modifié (August 2022)
    Probablement remarquer qu'on a $\displaystyle \int_{-1}^{1} \frac{\arccos(x)}{x^2+x+1}dx=\frac{\pi}{2}\int_0^1 \frac{1}{x^2+x+1}dx-\int_{-1}^{1} \frac{\arcsin(x)}{x^2+x+1}dx$
    (pas sûr que cela mène bien loin).
  • Modifié (August 2022)
    Tu as utilisé les propriétés de arccos, arcsin ? 
  • Le changement de variable $x=\cos\theta$ ainsi que le résultat renvoyé par Maple me laissent penser qu'il n'y a pas vraiment de réponse plus simple que l'intégrale elle-même.
  • Modifié (August 2022)
    Wolfram donne une primitive avec le dilogarithme 
    https://www.wolframalpha.com/input?i=integrate+arccos%28x%29%2F%28x%5E2%2Bx%2B1%29+dx
    peut-être dans le résultat y a du dilogarithme 
  • Mouais... 
  • WolframAlpha utilise des nombres complexes. Pour le fun, j'ai fait une DES du numérateur, dans $\C$ du coup, je trouve :
    $\dfrac{1}{x^2+x+1} = \dfrac{1}{(x-e^{2i\pi/3})(x-e^{-2i\pi/3})} = \dfrac{i\sqrt{3}}{3}\bigg(\dfrac{1}{x-e^{-2i\pi/3}} - \dfrac{1}{x-e^{2i\pi/3}}\bigg)$
    Si ma DES est correcte, peut-on en faire quelque chose d'utile ?
  • Modifié (August 2022)
    Bonjour
    J'ai trouvé $I=\dfrac{7 \pi ^2}{12 \sqrt{3}}-\dfrac{\pi  \arctan\left(\frac{4 \sqrt{2} \sqrt[4]{3} \left(26-15 \sqrt{3}\right)}{112 \sqrt{3}-193}\right)}{4 \sqrt{3}}  \approx  3.25879$
    mais le calcul est long. 
     
  • On peut montrer que $\displaystyle \int_{-1}^{1} \frac{\arcsin x}{1+x+x^2}dx=\frac{2}{\sqrt{3}}\int_0^\infty \frac{\arctan\left(\frac{3+x^2}{(1+x^2)\sqrt{3}}\right)}{1+x^2}dx-\frac{\pi^2}{3\sqrt{3}}$
    Et après on a envie de faire le changement de variable $y=\dfrac{3+x^2}{(1+x^2)\sqrt{3}}$
    (les bornes seront $\displaystyle \int_{\frac{1}{\sqrt{3}}}^{\sqrt{3}}$ )
  • Modifié (August 2022)
    La valeur close donnée par Bd2017, m'a suggéré une idée: Supposons qu'une valeur close est de la forme $\dfrac{\pi\arctan \alpha}{\sqrt{3}}$ Avec $\alpha$ un nombre algébrique de degré $4$*. ? p 300;
    J=intnum(x=-1,1,aos(x)/(1+x+x^2));
    r=tan(J*sqrt(3)/Pi);
    lindep([r,1,r^2,r^3,r^4]) realprecision = 308 significant digits (300 digits displayed) %2 = [96, -33, 6, 96, 23]~On trouve une valeur $\alpha$ qui serait un nombre algébrique de degré $4$*: si on cherche un nombre algébrique de degré inférieur à $4$ cela échoue.
    PS. Dans le calcul précédent je prends la tangente d'un angle plus grand que $\dfrac{\pi}{2}$. Pour éviter ce problème il vaut mieux chercher une valeur close sous la forme: $\dfrac{2\pi\arctan \alpha}{\sqrt{3}}$
    Avec $\alpha$ un nombre algébrique de degré $4$ p 400;J=intnum(x=-1,1,acos(x)/(1+x+x^2));
    r=tan(J*sqrt(3)/Pi*1/2);
    lindep([1,r,r^2,r^3,r^4]) %9 = [-3, 24, -30, 24, -11]~
  • Modifié (August 2022)
    Ce qui fait qu'une valeur close serait $\displaystyle \frac{2\pi}{\sqrt{3}}\arctan\left(\frac{1}{11} \left(6 + \sqrt{3} +2 \sqrt{14 \sqrt{3} - 15}\right)\right)$
  • Modifié (August 2022)
    Si on fait le changement de variable suggéré plus haut on obtient :
    $\displaystyle \int_0^\infty \frac{\arctan\left(\frac{3+x^2}{(1+x^2)\sqrt{3}}\right)}{1+x^2}dx=\frac{3^{\frac{1}{4}}}{2}\int_{\frac{1}{\sqrt{3}}}^{\sqrt{3}}\frac{\arctan u}{\sqrt{\sqrt{3}-u}\sqrt{u\sqrt{3}-1}}du$
    Après on a envie d'enchaîner avec le changement de variable $y=\dfrac{1}{u}$ mais on arrivera pas à faire disparaître, hélas, l'arctangente du fait des racines carrées au dénominateur on va récupérer un facteur $1/u$
    Cela me semble plus prometteur d'introduire $\displaystyle F(a)=\int_{\frac{1}{\sqrt{3}}}^{\sqrt{3}}\frac{\arctan(au)}{\sqrt{\sqrt{3}-u}\sqrt{u\sqrt{3}-1}}du$ du fait que la dérivée de l'intégrande par rapport à la variable $a$ semble avoir une primitive pas trop laide quand on intègre en fonction de $u$.
  • Modifié (August 2022)
    Fin de partie a dit : https://les-mathematiques.net/vanilla/index.php?p=/discussion/comment/2372984/#Comment_2372984
    On peut montrer que $\displaystyle \int_{-1}^{1} \frac{\arcsin x}{1+x+x^2}dx=\frac{2}{\sqrt{3}}\int_0^\infty \frac{\arctan\left(\frac{3+x^2}{(1+x^2)\sqrt{3}}\right)}{1+x^2}dx-\frac{\pi^2}{3\sqrt{3}}$
    Comment on arrive à cette égalité ?Merci
  • Changement de variable $u=\arcsin x$ puis intégration par parties, puis changement de variable $x=\cot x$.
  • Modifié (August 2022)
    Une méthode sans doute plus simple: \begin{align}J&=\int_{-1}^1 \frac{\arccos x}{1+x+x^2}dx\\ &=\int_{0}^1 \frac{\frac{\pi}{2}+\arcsin x}{1-x+x^2}dx+\int_0^1 \frac{\arccos x}{1+x+x^2}dx\\ &=\frac{\pi^2}{3\sqrt{3}}+\int_0^1 \frac{\arcsin x}{1-x+x^2}dx+\int_0^1 \frac{\arccos x}{1+x+x^2}dx\\ &\overset{u=\frac{1-x}{1+x}}=\frac{\pi^2}{3\sqrt{3}}+\int_0^1 \frac{\pi-4\arctan(\sqrt{u})}{1+3u^2}du+\int_0^1 \frac{4\arctan(\sqrt{u})}{3+u^2}du\\ &=\frac{2\pi^2}{3\sqrt{3}}-\int_0^1 \frac{4\arctan(\sqrt{u})}{1+3u^2}du+\int_0^1 \frac{4\arctan(\sqrt{u})}{3+u^2}du\\ &=\frac{2\pi^2}{3\sqrt{3}}-4\int_0^1 \frac{\arctan(\sqrt{u})}{1+3u^2}du+4\bigg(\int_0^\infty \frac{\arctan(\sqrt{u})}{3+u^2}du-\underbrace{\int_1^\infty \frac{\arctan(\sqrt{u})}{3+u^2}du}_{z=\frac{1}{u}}\bigg)\\ &=\frac{2\pi^2}{3\sqrt{3}}-4\Big(\int_0^1 \frac{\arctan(\sqrt{u})}{1+3u^2}du+\int_0^1 \frac{\arctan\big(\frac{1}{\sqrt{z}}\big)}{1+3z^2}dz\Big)+4\int_0^\infty \frac{\arctan(\sqrt{u})}{3+u^2}du\\ &=4\int_0^\infty \frac{\arctan(\sqrt{u})}{3+u^2}du\\ \end{align} Après on a envie d'introduire $\displaystyle F(a)=\int_0^\infty \frac{\arctan(a\sqrt{u})}{3+u^2}du$
  • @ Fin de partie tu es un génie pour les calculs d’intégrales 
  • Modifié (August 2022)
    @etanche: J'avais déjà calculé un truc comme ça* car c'est à cette occasion que j'ai rencontré les identités, pour $0\leq x\leq 1$:
    \begin{align} \arcsin\left(\frac{1-x}{1+x}\right)&=\frac{\pi}{2}-2\arctan\left(\sqrt{x}\right)\\\arccos\left(\frac{1-x}{1+x}\right)&=2\arctan\left(\sqrt{x}\right)\end{align}
    Le résultat final est donc: $\boxed{\displaystyle \int_{-1}^{1} \frac{\arccos(x)}{x^2+x+1}dx=\dfrac{4\pi^2}{3\sqrt{3}}-\dfrac{2\pi}{\sqrt{3}}\arctan\left(\sqrt{3+2\sqrt{3}}\right)}$ *: je m'en suis souvenu quand j'ai pensé à faire le changement de variable $u=\dfrac{1-x}{1+x}$
    PS.
    Est-ce que $\sqrt{3+2\sqrt{3}}$ est la tangente d'un angle remarquable? Wolfy ne semble pas de cet avis.
  • Modifié (August 2022)
    ? \p 200
    r=atan(sqrt(3+2*sqrt(3)))/Pi;
    lindep([1,r,r^2,r^3,r^4])
    %3 = [8821489149531819365978870294898, -61061596227811700824488235359158, 96114185557034829361314483137396, -44239872228442594444941441125459, 139731881664842207378918565766024]~
    Ce qui suggère qu'on n'arrivera pas à exprimer l'arctangente en un produit de $\pi$ par une racine d'un polynôme de degré $4$
    (donc en particulier comme un rationnel fois $\pi$)
  • A noter que sauf erreur on a  $\tan^{-1}\left(\sqrt{3+2\sqrt{3}}\right)=\cos^{-1}\left(\frac{\sqrt{3}-1}{2}\right)$.

  • Modifié (August 2022)
    Qu’en est-il pour $$K=\int_{-1}^{1} \frac{\arccos(x)}{P(x)} dx\qquad\text{ou}\qquad L=\int_{0}^{1} \frac{\arccos(x)}{P(x)}dx,$$avec $P(x)$ d’autres polynômes autres que $x^2+1,\ x^2+x+1$ qui ne s’annulent pas sur $[-1;1]$ peut-on trouver la valeur de l’intégrale ?
    par exemple $P(x)=x^4+1$ pour $K$ ou $L$
    $P(x)=x^3+1$ pour $L$ ?
  • Modifié (August 2022)
    \[P\left(\frac{1-x}{1+x}\right)=\frac{\left( c-b+a\right)  {{x}^{2}}-2 \left( c-a\right)  x+c+b+a}{{{\left( x+1\right) }^{2}}},\]
    avec $P(x)=a+bx+cx^2$.
    Dans les calculs que j'ai faits plus haut on avait $a=c$ ce qui supprime le terme en $x$.
  • On peut peut-être arriver à calculer $\displaystyle \int_0^1 \frac{\arccos x}{1+x+x^2+x^3}dx=\int_0^1 \frac{(1+x)\arctan\left(\sqrt{x}\right)}{1+x^2}dx$
  • Modifié (August 2022)
    Merci
    @etanche : ce serait courtois de référencer les exos que tu proposes pour qu'on puisse savoir si on a affaire à un exo inventé, une question ouverte, un post d'un autre forum, etc. Ca aide un peu à organiser les éventuelles recherches de ces énoncés bien intéressants pour la plupart.
    Merci (ça pour le coup, on ne peut que saluer cette marque de politesse bien présente à chacun de tes messages elle).
  • Modifié (August 2022)
    @Jlapin: Je me suis posé la même question, à savoir, s'il n'y a pas de valeur donnée pour l'intégrale est-ce que cela vaut la peine d'y consacrer du temps alors qu'il se pourrait que l'intégrale n'ait aucune valeur remarquable.
    Pour la décharge d'Etanche on remarquera que les calculs d'intégrales qu'il a postés sont quasiment tous issus du journal American mathematical monthly  et les dates à laquelle il poste ces problèmes correspondent aux dates de  livraisons de problèmes sur le site qui publie l'AMM*.
    Je lui ai demandé par MP d'où venait ce problème et la réponse qu'il m'a donnée me fait penser qu'il en ignorait réellement l'origine.
    *: et si on n'a pas d'accès à ces problèmes on attend que l'excellent Tauraso ait traité ledit problème pour avoir la confirmation que c'était bien un problème de l'AMM.
  • Modifié (August 2022)
    @ JLapin Un ami me l’a envoyé.
  • Modifié (August 2022)
    Donc, Etanche, tu as des amis qui te font faire les exercices de l'AMM, et tu les fais faire par les costauds du forum. Ce n'est pas hors charte, ça ???
  • Modifié (August 2022)
    Je ne vois vraiment pas pourquoi vous vous étonnez
    C'est le jeu
    Etanche est Etanche (et pas quelqu'un d'autre)
    Pour rappel concernant ses sources
    On parle d'étanchéité dans le domaine de la communication et de l'information quand des moyens de protection contre la diffusion sont mis en place. Des accords de confidentialité sont souvent mis en place pour garantir l'étanchéité de l'information de l'intérieur vers l'extérieur des entreprises.
  • Modifié (August 2022)
    Cet exercice avec des questions intermédiaires peut faire un bon TD maths spé en deux parties 
    avec en première partie le calcul de 
    https://math.stackexchange.com/questions/2635503/definite-integral-int-11-frac-arccos-leftx-right1x2dx?noredirect=1


  • Modifié (August 2022)
    On imagine bien que tu as des sources du type de celles que tu cites. Il me semblerait normal de les donner systématiquement quand tu proposes l'exercice. Je ne comprends pas pourquoi cette règle élémentaire de courtoisie n'est pas appliquée à chaque fois. Quant à ton ami qui t'envoie des problèmes d'un journal mathématique sans citer sa source, tu pourrais également lui demander de les citer la fois prochaine.
    Merci.
  • Modifié (August 2022)
    JLapin: Qu'est-ce que cela apporte si la source est "un ami me l'a envoyé" ? La prochaine fois on espère que son ami donnera sa source.
    C'est bien d'indiquer un numéro de problème de l'AMM pour faciliter des recherches futures sur le forum.
  • Modifié (September 2022)
    Bonjour
    Calculer $$\int_{-1}^1 \frac{\arccos(x)}{x^2+x+1}  dx.$$
    Une intégrale diabolique  (AMM  12344 ).
    Le 😄 Farceur


  • Modifié (September 2022)
    PS.
    Signaler le numéro d'un problème de l'AMM ce n'est pas seulement pour donner la source mais aussi afin qu'on puisse retrouver un tel énoncé de problème sur le forum aisément.
  • OK merci, j'étais absent du forum jusqu'à fin Août. IL ya aussi cette jolie intégrale  numéro 12332 
    https://www.mat.uniroma2.it/~tauraso/AMM/amm.html
    Le 😄 Farceur


  • Je me demande si une raison est cachée du fait que etanche ou son ami ne donne presque jamais la source s'elle provient d'un problème AMM en cours.  Peut-être ils en profitent des solutions pour se faire une réputation à l 'AMM en envoyant comme solutions le fruit des efforts des membres.
    Le 😄 Farceur


  • Modifié (September 2022)
    @Gebrane: Procéder de la sorte serait minable. Cela ne rapporte rien d'envoyer une solution à l'AMM même pas quelques mois d'abonnement gratuit. >:)
    J'ai déjà résolu le problème 12332 si je me souviens bien.
    PS.
    Si Etanche propose un calcul d'intégrale vers le début d'un mois tu peux être à peu près sûr que c'est un problème de l'AMM. Les listes de problèmes sont publiées en début de mois, tout du moins c'est la tendance du moment sur le site de l'AMM. Dans la semaine qui suit, Tauraso l'aura calculée. >:)
  • L'intégrale du problème 12332  se  calcule  avec la somme  résidus de  la fonction qui sont $\dfrac{16 i}{\pi ^3 (2 k+1)^3} $ Le résultat en découle directement 

     
  • @Bd2017: C'est moche comme calcul.
  • Modifié (September 2022)
    @FDP  Oui c'est trop simple.  Tu calcules le résidu  en $x_k=i(\pi/2+k\pi)$  et c'est terminé.  Si c'est moche, cela veut dire que tu peux faire le calcul avec encore moins de  calculs ?  
    Tu peux faire 2 IPP et puis faire le calcul mais c'est plus calclulatoire.

     
  • @bd2017:  On peut faire ce calcul avec des techniques élémentaires d'analyse réelle.
  • @FDP Oui je m'en doute très bien.  Par ailleurs tu fais 2 IPP et tu finis avec Wolfram.  Cette partie que sait faire Wolfram est un peu galère à la main à  mon avis. Mais  les manipulations techniques te sont plus familières. Alors montre un peu...le calcul . Il y a toujours à apprendre dans tes façons de faire, mais je n'ai pas encore vu la systématique ...
    Ceci étant dit, l'application du théorème des résidus n'est pas moche du tout ...elle est efficace...
     
  • Modifié (September 2022)
    On montre que \begin{align}\int_0^\infty \frac{\tanh^2 x}{x^2}dx&\overset{\text{IPP}}=\int_0^\infty \frac{2\tanh x}{x\cosh^2 x}dx\\ &\overset{u=\tanh x}=2\int_0^1 \frac{u}{\text{arctanh } u}du\\ &=-4\int_0^1 \frac{u}{\ln\left(\frac{1-u}{1+u}\right)}du\\ &\overset{z=\frac{1-u}{1+u}}=8\int_0^1 \frac{z-1}{(1+z)^3\ln z}dz\\ &=8\int_0^1\int_0^1 \frac{z^t}{(1+z)^3}dtdz\\ &\overset{x=1-t}=8\int_0^1\int_0^1 \frac{z^{1-x}}{(1+z)^3}dxdz\\ &=4\int_0^1\int_0^1 \frac{z^{1-x}+z^x}{(1+z)^3}dxdz\\ &=4\int_0^1 \text{B}(2-x,1+x)dx \end{align}
    Je vous laisse finir (utilisation des propriétés de $\Gamma$ et formule des compléments etc). $B$ est la fonction Bêta d'Euler.
  • Modifié (September 2022)
    Merci @Fin de partie , mais je cherche une preuve sans passer par l'intégrale double, je vais y réfléchir à partir de 
    $\int_0^1 \frac{z-1}{(1+z)^3\ln z}dz$
    Le 😄 Farceur


  • @gebrane: Tauraso a sûrement une preuve avec un calcul de série mais je doute qu'on puisse arriver à se dispenser d'intégrale double,  de dérivation sous le signe intégral, de série.
  • La formule mentionnée ici est parfois très utile.
  • @Fin de partie J'ai  sous la main le même exercice
    Calculer $\int_0^1 \frac{z-1}{(1+z)^3\ln z}dz $
    sans solution qui a pour objectif de calculer ledit intégrale par Frullani. Mystère 
    Le 😄 Farceur


  • @Gebrane: Frullani camoufle l'utilisation d'intégrale double (Je ne crois pas qu'on puisse démontrer ladite formule sans intégrale double, dérivation sous le signe intégral)
  • Ne soit pas pessimiste, je peux te montrer comment utiliser Frullani pour  
    $$\int_0^1 \frac{z-1}{(1+z)\ln z}dz$$
    Le 😄 Farceur


Connectez-vous ou Inscrivez-vous pour répondre.
Success message!